Finding three orders of integration for a triple integral over unusual region

In summary, the problem involves finding two orders of integration for a given region D in terms of x_1, x_2, and x_3. One possible approach is to integrate first with respect to x_3, then x_2, and finally x_1. Another approach is to rotate the coordinate system in the x_1 x_2-plane by 45 degrees. However, the integral must be written in terms of the original coordinates x_1, x_2, and x_3. The conversation also discusses redefining the region D in order to make the limits of x_3 dependent only on x_1. This approach is explored by splitting D into smaller regions and redefining them accordingly.
  • #1
mistahkurtz
4
0

Homework Statement


fvwwS.png


2. The attempt at a solution

It's not hard to find two orders of integration.

(1) Integrate first with respect to [tex]x_3[/tex], then with respect to [tex]x_2[/tex], and then with respect to [tex]x_1[/tex], by dividing D into two regions:

[tex]D = \{x \in R^3 \mid -1 \leq x_1 < 0, -\sqrt{1-x_1^2} \leq x_2 \leq \sqrt{1-x_1^2}, -\sqrt{5 - x_1 - x_2} \leq x_3 \leq \sqrt{5 - x_1 - x_2}\}[/tex] [tex]\cup \{x \in R^3 \mid 0 \leq x_1 \leq 1, -\sqrt{1-x_1^2} \leq x_2 \leq 1-x_1, -\sqrt{5 - x_1 - x_2} \leq x_3 \leq \sqrt{5 - x_1 - x_2}\}[/tex]

(2) Integrate first with respect to [tex]x_3[/tex], then with respect to [tex]x_1[/tex], and then with respect to [tex]x_2[/tex], by dividing D into three regions

[tex]D = \{x \in R^3 \mid -1 \leq x_2 < 1, -\sqrt{1-x_2^2} \leq x_1 < 0, -\sqrt{5 - x_1 - x_2} \leq x_3 \leq \sqrt{5 - x_1 - x_2}\}[/tex] [tex]\cup \{x \in R^3 \mid 0 \leq x_2 \leq 1, 0 \leq x_1 \leq 1-x_2, -\sqrt{5 - x_1 - x_2} \leq x_3 \leq \sqrt{5 - x_1 - x_2}\}[/tex] [tex]\cup \{x \in R^3 \mid -1 \leq x_2 \leq 0, 0 \leq x_1 \leq \sqrt{1-x_2^2}, -\sqrt{5 - x_1 - x_2} \leq x_3 \leq \sqrt{5 - x_1 - x_2}\}[/tex]

(3) I'm having difficulty finding how to define D so that I can integrate first with respect to [tex]x_2[/tex] or [tex]x_1[/tex], then with respect to [tex]x_3[/tex], and last with respect to [tex]x_1[/tex] or [tex]x_2[/tex]. The problem is that the limits of [tex]x_3[/tex] depend on both [tex]x_1[/tex] and [tex]x_2[/tex], and I can't seem to manipulate the inequalities correctly to give me what I want.

I tried following the method used in Example 5 here (http://www.math.umn.edu/~nykamp/m2374/readings/tripintex/) in order to redefine just the subset of D for which [tex]x_1[/tex] and [tex]x_2[/tex] are non-negative, let's call it [tex]D_1[/tex].

[tex]D_1 = \{x \in R^3 \mid 0 \leq x_1 \leq 1, 0 \leq x_2 \leq 1-x_1, 0 \leq x_3 \leq \sqrt{5 - x_1 - x_2}\}[/tex]

Since it is also true that [tex]0 \leq x_3 \leq \sqrt{5 - x_1}[/tex] and [tex]0 \leq x_2 \leq 5 -x_1 - x_3^2[/tex], that website recommends defining

[tex]D_1 = \{x \in R^3 \mid 0 \leq x_1 \leq 1, 0 \leq x_3 \leq \sqrt{5 - x_1}, 0 \leq x_2 \leq 5 -x_1 - x_3^2\}[/tex]

but I don't think that's right. If that were correct, (0.5,4.5,0) would be a point in [tex]D_1[/tex], which it isn't since if [tex]x_1=1[/tex], then it must be the case that [tex]0 \leq x_2 \leq 0.5[/tex]. So basically, I'm stumped. :(
 
Last edited:
Physics news on Phys.org
  • #2
Have you tried rotating the coordinate system in the [tex]x_1 x_2[/tex]-plane by 45 degrees?
 
  • #3
ystael said:
Have you tried rotating the coordinate system in the [tex]x_1 x_2[/tex]-plane by 45 degrees?

I thought about doing that. But wouldn't the integral no longer be "an integral of a function f(x) over the region D," as the problem explicitly states, but rather "an integral of a function [tex]f(\Phi(x))[/tex] over the region [tex]\phi(D)[/tex]" where [tex]\Phi(x)[/tex] is the linear transformation that rotates the coordinate system in the [tex]x_1 x_2[/tex] plane?

EDIT: Yeah, I asked my professor (who wrote the problem), and he says the integral must be written in terms of the coordinates [tex]x_1, x_2, x_3[/tex].
 
Last edited:
  • #4
Well, I think I've progressed a bit towards finding an answer. I think I know how to redefine [tex]D_1[/tex]. This

[tex]D_1 = \{x \in R^3 \mid 0 \leq x_1 \leq 1, 0 \leq x_3 \leq \sqrt{5 - x_1}, 0 \leq x_2 \leq 5 -x_1 - x_3^2\}[/tex]

is wrong because the upper limit of [tex]x_2[/tex] should be the minimum of [tex]1 - x_1[/tex] and [tex]5 - x_1 - x_3^2[/tex] not simply [tex]5 - x_1 - x_3^2[/tex]. Then, since [tex] 1 - x_1 \leq 5 - x_1 - x_3^2[/tex] if and only if [tex]x_3 \leq 2[/tex], we have

[tex]D_1 = \{x \in R^3 \mid 0 \leq x_1 \leq 1, 2 < x_3 \leq \sqrt{5 - x_1}, 0 \leq x_2 \leq 5 -x_1 - x_3^2\} \cup \{x \in R^3 \mid 0 \leq x_1 \leq 1, 0 \leq x_3 \leq 2, 0 \leq x_2 \leq 1-x_1\}[/tex]

Is my reasoning correct? Should I continue doing this - splitting up D into smaller regions and redefining them so the limits of [tex]x_3[/tex] depend only on [tex]x_1[/tex]?
 
Last edited:

Related to Finding three orders of integration for a triple integral over unusual region

1. What are the three orders of integration for a triple integral?

The three orders of integration for a triple integral are the innermost, middle, and outermost integrals. These correspond to the x, y, and z variables respectively.

2. How do you determine the order of integration for a triple integral?

The order of integration for a triple integral is determined by the shape and orientation of the region of integration. Generally, the innermost integral corresponds to the variable with the smallest range of values, followed by the middle and outermost integrals.

3. What is an unusual region in the context of triple integrals?

An unusual region in the context of triple integrals refers to a three-dimensional region that does not have a simple geometric shape, such as a cube or sphere. It may have curved or irregular boundaries that require careful consideration when choosing the order of integration.

4. How do you approach finding the three orders of integration for an unusual region?

To find the three orders of integration for an unusual region, it is helpful to first visualize the region and its boundaries. Then, determine the range of values for each variable and choose the order of integration accordingly. It may also be helpful to break down the region into simpler sub-regions and find the orders of integration for each sub-region separately.

5. Can the order of integration change for different regions?

Yes, the order of integration can change for different regions. It is important to carefully analyze the boundaries and range of values for each variable in order to determine the appropriate order of integration for a specific region. This may vary depending on the shape and orientation of the region.

Similar threads

  • Calculus and Beyond Homework Help
Replies
2
Views
681
Replies
3
Views
766
  • Calculus and Beyond Homework Help
Replies
15
Views
1K
  • Advanced Physics Homework Help
Replies
1
Views
818
  • Calculus and Beyond Homework Help
Replies
9
Views
562
  • Set Theory, Logic, Probability, Statistics
Replies
6
Views
765
  • Calculus and Beyond Homework Help
Replies
9
Views
998
  • Topology and Analysis
Replies
1
Views
989
Replies
22
Views
2K
  • Calculus and Beyond Homework Help
Replies
27
Views
2K
Back
Top